Difference between revisions of "2016 AMC 10B Problems/Problem 18"

(correct answer is E)
m (Solution)
Line 8: Line 8:
 
Factorize <math>345=3\cdot 5\cdot 23</math>.
 
Factorize <math>345=3\cdot 5\cdot 23</math>.
  
Suppose we take an odd number <math>k</math> of consecutive integers, centred on <math>m</math>. Then <math>mk=345</math> with <math>\tfrac12k<m</math>.
+
Suppose we take an odd number <math>k</math> of consecutive integers, centered on <math>m</math>. Then <math>mk=345</math> with <math>\tfrac12k<m</math>.
 
Looking at the factors of <math>345</math>, the possible values of <math>k</math> are <math>3,5,15,23</math> centred on <math>115,69,23,15</math> respectively.
 
Looking at the factors of <math>345</math>, the possible values of <math>k</math> are <math>3,5,15,23</math> centred on <math>115,69,23,15</math> respectively.
  

Revision as of 14:22, 18 June 2016

Problem

In how many ways can $345$ be written as the sum of an increasing sequence of two or more consecutive positive integers?

$\textbf{(A)}\ 1\qquad\textbf{(B)}\ 3\qquad\textbf{(C)}\ 5\qquad\textbf{(D)}\ 6\qquad\textbf{(E)}\ 7$

Solution

Factorize $345=3\cdot 5\cdot 23$.

Suppose we take an odd number $k$ of consecutive integers, centered on $m$. Then $mk=345$ with $\tfrac12k<m$. Looking at the factors of $345$, the possible values of $k$ are $3,5,15,23$ centred on $115,69,23,15$ respectively.

Suppose instead we take an even number $2k$ of consecutive integers, centred on $m$ and $m+1$. Then $k(2m+1)=345$ with $k\le m$. Looking again at the factors of $345$, the possible values of $k$ are $1,3,5$ centred on $(172,173),(57,58),(34,35)$ respectively.

Thus the answer is $\textbf{(E) }7$.

See Also

2016 AMC 10B (ProblemsAnswer KeyResources)
Preceded by
Problem 17
Followed by
Problem 19
1 2 3 4 5 6 7 8 9 10 11 12 13 14 15 16 17 18 19 20 21 22 23 24 25
All AMC 10 Problems and Solutions

The problems on this page are copyrighted by the Mathematical Association of America's American Mathematics Competitions. AMC logo.png